LSAT and Law School Admissions Forum

Get expert LSAT preparation and law school admissions advice from PowerScore Test Preparation.

 silent7706
  • Posts: 42
  • Joined: Mar 26, 2019
|
#65364
Hi,

I just don't think (A) does much to strengthen the conclusion "...a dramatic increase in the number of people over the age of 65."

It's easy to construct a scenario where you have more younger people below 18, while no increase of the number people age above 65. For example, all you need is to increase number of old people from 57 - 64.9(just below 65), while having some more people younger than 18. Your average population age can still increase from 52 to 57 while having no effect on the number of people who are 65+.

I hope I did not miss something big here because that will be unfortunate since I deal with numbers on a daily basis as an accountant, haha.
 Adam Tyson
PowerScore Staff
  • PowerScore Staff
  • Posts: 5153
  • Joined: Apr 14, 2011
|
#65365
Looks to me like you are trying to prove that there are more people over 65, silent7706, rather than simply help that conclusion look a little better, which is all you need for a Strengthen answer. While you're right that the correct answer doesn't prove the conclusion, it certainly helps a lot by eliminating an obvious possible alternate cause for the increased average. Don't set the bar too high on these questions! Anything that helps, helps! Does answer A make the conclusion seem a bit more plausible, a bit more likely to be true? That's the standard to apply, and A does that while none of the other answers matter at all.
 silent7706
  • Posts: 42
  • Joined: Mar 26, 2019
|
#65403
Thank you Adam. That total makes sense to me now.

Get the most out of your LSAT Prep Plus subscription.

Analyze and track your performance with our Testing and Analytics Package.